You are on page 1of 6

MATH 163, Spring 2020

Script 16

Definition 16.1. Let (an )∞


n=1 be a sequence of real numbers. Then the formal sum


X
an
n=1

is called an infinite series. More generally, we may start the series at N0 as long as an is defined

for all n ≥ N0 .

We define the sequence of partial sums (pn ) of the series by

n
X
pn = a1 + · · · + an = ai .
i=1

We say that the series converges if there exists L ∈ R such that lim pn = L. When this is the
n→∞
case, we write this as

X
an = L,
n=1

and we say that L is the sum of the series. When there does not exist such an L, we say that the

series diverges.

X
Lemma 16.2. Let (an )∞
n=0 be a sequence of real numbers. Let N0 ∈ N. Then an converges if,
n=0

X
and only if, an converges.
n=N0


X ∞
X
Proof. (→) For the forward direction: Let an converge to L. As such, pN0 + an = L,
n=0 n=N0

X
which means that an = L − pN0 = L0 . L0 must exist because pN0 is finite, and therefore,
n=N0

X ∞
X
an = L0 . As such, by Script 16.1, an converges.
n=N0 n=N0

1

X
(←) For the reverse direction: Let an converge to L. As such, since pN0 is finite, then
n=0

X ∞
X ∞
X
pN0 + an = an = L + pN0 = L0 . As such, this means that an converges to L0 . Therefore,
n=N0 n=0 n=0

X
by Script 16.1, an converges.
n=0
∞  
X 1 1
Exercise 16.3. Prove that − converges. What is its sum?
n n+1
n=1

Proof. These sums can be rewritten as:

k k k+1
X X 1 X1 1 1
= − = − .
n n 1 k+1
n=1 n=1 n=2

1
As such, because limk→∞ k+1 = 0, the sequence of partial sums, and thus the infinite sum above

converges to 0.

X
Theorem 16.4. If an converges, then lim an = 0.
n→∞
n=1

Proof. If the infinite sum of (an ) converges, it must then follow that the partial sums must converge,

and so it can be written that limn→∞ pn+1 = limn→∞ pn . It then follows from that limn→∞ pn+1 −

pn = limn→∞ pn+1 − limn→∞ pn = limn→∞ an+1 = limn→∞ an = 0.

The converse of this theorem is however not true, as we see in Theorem 16.6.

X
Theorem 16.5. A series an converges if, and only if, for all  > 0, there is some N ∈ N such
n=1
n
X
that ak < , ∀n > m ≥ N.


k=m+1

Proof. (→) For the forward direction: It follows that the infinite sum converges if and only if the

sequence of partial sums pn = ni=1 ai converges. It follows from Script 14.31 that if this sequence
P

of partial sums (pn ) converges, then it is a Cauchy sequence, and thus for all  > 0, there must

exist some N ∈ N such that |pn − pm | = | nk=m+1 ak | < , for all n > m ≥ N .
P

(←) For the reverse direction: For all  > 0, there is some N ∈ N such that |pn − pm | < , for

all n > m ≥ N . As such, for all n, m ≥ N , if n = m, then |pn − pm | = 0 < . If n 6= m, WLOG let

2
n > m, and it thus it follows that |pn − pm | < , which implies that pn is a Cauchy sequence and

thus a convergent sequence, and thus the infinite sum ∞


P
n=1 an must converge.
∞ 2N
X 1 1
≥ 12 . Note that you must prove the
P
Theorem 16.6. The series diverges. (Hint: n
n n=N +1
n=1
hint if you use it.)

Proof. To first prove the hint: It follows that for all n ∈ [N + 1, 2N ] such that n ∈ N, that n ≤ 2N ,
1
≤ n1 . Thus, it must follow that 2N 1 1 1
P
and thus that 2N m=N +1 n ≥ N · 2N = 2 .

Thus, from the hint, the series ∞ 1 1


P
n=1 n cannot converge, as for  = 4 , for all N ∈ N, it follows

that 2N > N + 1 ≥ N , and that | nk=m+1 ak | ≥ 21 6<  = 14 . Thus, this series must diverge.
P

Theorem 16.7. Let −1 < x < 1. Then,


X 1
xn = .
1−x
n=0

1
Proof. It should be shown that the sequence partial sums (pn ) of xn converge to 1−x .

Using the fact that pn = 1 + x + x2 . . . + xn = ∞ n 2 3 n+1 =


P
n=0 x , and so x · pn = x + x + x . . . + x
n+1
pn − 1 + xn+1 . As such, pn = 1−x
1−x , and so limn→0 pn =
1
1−x since |x| < 1, and so xn+1 approaches
P∞ n 1
0. Therefore, n=0 x = 1−x by Script 16.1.

P ∞
P
Theorem 16.8. If an = L and bn = M and c ∈ R, then
n=1 n=1


X
(an + bn ) = L + M, and
n=1
X∞
(c · an ) = c · L.
n=1

Proof. (a) Omitted.

(b) Let ∞
P
n=1 (an ) = L, c ∈ R, and  > 0. By Script 16.5, there must exist an N ∈ N such that

| ∞ 
P
k>m+c (ak )| < |c|+1 such that n > m ≥ N . Now, consider the following few equations:


X
| (c · an )| =
k=m+1

3
|c · (am+1 + . . . an )| =

|c||am+1 + · + an | =

X 
|c|| (ak )| < |c| · <
|c| + 1
k=m+1

P∞ P∞
As such, by Script 16.5, n=1 (c · an ) converges. Now, let pn = a1 + · · · + c · an = i=1 (c · ai ).

Then, by Script 16.1, limn→∞ pn = L, and by Script 15.6(a), an = c converges to c. As

such, by Script 15.5, limn→∞ c = c, and so limn→∞ c · pn = c · L. Therefore, by Script 16.1,


P∞
n=1 (c · an ) = c · L.


X ∞
X
Definition 16.9. We say that the series an converges absolutely if the series |an | converges.
n=1 n=1

X
Lemma 16.10. A series an with all an ≥ 0 converges if and only if its sequence of partial sums
n=1
is bounded.

Proof. (→) For the forward direction: The forward direction follows from 15.15, as for (pn ) to

converge, it must be bounded.

(←) For the reverse direction: For a sequence (an ) with all terms an ≥ 0, it must follow that

the sequence of partial sums (pn ) is increasing. Thus, if by 15.16, (pn ) is bounded and increasing,
X∞
it must converge to its supremum and thus an must exist.
n=1

X ∞
X
Theorem 16.11. If an converges absolutely then an converges and
n=1 n=1

∞ ∞

X X
an ≤ |an |.



n=1 n=1

(Hint: Let bn = max{0, an }, and cn = min{0, an }. Then an = bn + cn .)


P∞
Proof. Because n=1 an
converges absolutely, it must follow that for any  > 0, there exists N

such that if m ≥ n ≥ N , it follows then that | m


P
n+1 |ai | < . Because ai ≤ |ai | for all i ∈ N, it

must follow that | n+1 ai | ≤ n+1 |ai | < . Thus, ∞


Pm Pm P
n=1 an must converge.

4
P P P P
Taking the definitions of bn and cn from the hint, then an = bn + cn , and that |an | =

|an | − an = −2 cn ≥ 0, and thus | ∞


P P P P P P P∞
bn − cn . As such, n=1 an | ≤ n=1 |an |.

Theorem 16.12. Let (an ) be a decreasing sequence of positive numbers such that lim an = 0.
n→∞

X
Then, (−1)n+1 an converges.
n=1

Proof. Omitted.

Theorem 16.13. Let (cn ) be a sequence of positive numbers and let (an ) be a sequence such that

P ∞
P
|an | ≤ cn for all n ≥ N0 , where N0 is some fixed integer. If cn converges, then an converges.
n=1 n=1

P
Proof. Since cn converges, it follows by definition that its partial sequence of partial sums is
n=1
bounded, and so for some M ∈ R, 0 ≥ ni=1 ci ≤ M for all n ∈ N. Also, because 0 ≤ |an | ≤ cn for
P

all n ∈ N, then 0 ≤ ni=1 |ai | ≤ ni=1 ci ≤ M . Thus, it follows that ∞


P P P
n=1 |an | converges, and thus
P∞
that n=1 an converges.

Lemma 16.14. Suppose that (bn ) is a sequence of non-negative numbers with lim bn = L, where
n−→∞
1+L
L < 1. Prove that there is some N ∈ N such that 0 ≤ bn < , for all n ≥ N.
2

Proof. Because (bn ) is a sequence of non-negatives, it must follow that L ≥ 0 (or else there would

be contradiction with delta epsilon).


1−L
Thus, we know from epsilon delta that for  = 2 , there must exist N ∈ N such that for all
1+L
n ≥ N , |bn − L| < , which would imply that 0 ≤ bn < 2 .

an+1
Theorem 16.15. Let (an ) be a sequence such that lim
exists. Then,
n→∞ an


an+1 X
(a) If lim
< 1, then an converges.
n→∞ an
n=1

Proof. There must exist some N ∈ N such that for all n ≥ N , | an+1
an | <
1+L
2 . In other terms,

that |an+1 | < |an | 1+L 1+L n−N


2 . Thus, it follows that for all n ≥ N , then |an | ≤ bn = ( 2 ) aN .
1+L
Also, (bn ) converges, as it is a geometric sequence with a common ratio of 2 < 1, and thus
P∞
n=1 an converges.

5

an+1 X
(b) If lim
> 1, then an diverges.
n→∞ an
n=1

Proof. Omitted.

p
n
Theorem 16.16. Let (an ) be a sequence such that lim |an | exists. Then
n→∞

p ∞
X
n
a) If lim |an | < 1, then an converges.
n→∞
n=1

Proof. Omitted.

p ∞
X
n
b) If lim |an | > 1, then an diverges.
n→∞
n=1

p
n
Proof. Because L > 1, there exists N such that |an | > 1 for all n ≥ N , thus implying that

limn→∞ an 6= 0, and thus the series diverges.

Definition 16.17. For n ∈ N, we define the factorial of n to be the product of all natural numbers

less than or equal to n. We denote this by the formula

n! = n · (n − 1) · (n − 2) · · · · · 3 · 2 · 1.

By convention, we also set 0! = 1.

Exercise 16.18. Prove that



X 1
n!
n=0

converges. The number that it converges to is called e.

Proof. It follows by the ratio test that limn→∞ | an+1


an | = limn→∞
1
n+1 = 0 < 1, and thus the series

converges.

You might also like